\lecture{17}{2023-06-15}{} \subsection{Doob's Martingale Convergence Theorem} \begin{definition}[Stochastic process] A \vocab{stochastic process} is a collection of random variables $(X_t)_{t \in T}$ for some index set $T$. In this lecture we will consider the case $T = \N$. \end{definition} \begin{definition}[Previsible process] Consider a filtration $(\cF_n)_{n \ge 0}$. A stochastic process $(C_n)_{n \ge 1}$ is called \vocab[Stochastic process!previsible]{previsible}, iff $C_n$ is $\cF_{n-1}$-measurable. \end{definition} \begin{goal} What about a ``gambling strategy''? Consider a stochastic process $(X_n)_{n \in \N}$. Note that the increments $X_{n+1} - X_n$ can be thought of as the win or loss per round of a game. Suppose that there is another stochastic process $(C_n)_{n \ge 1}$ such that $C_n$ is determined by the information gathered up until time $n$, i.e.~$C_n$ is previsible. Think of $C_n$ as our strategy of playing the game. Then $C_n(X_n - X_{n-1})$ defines the win in the $n$-th game, while \begin{equation} Y_n \coloneqq \sum_{j=1}^n C_j(X_j - X_{j-1}) \label{eqn:cumulative-win-process} \end{equation} defines the cumulative win process. \end{goal} \begin{lemma} \label{lem:gambling-strategy} If $(C_n)_{n \ge 1}$ is previsible and $(X_n)_{n \ge 0}$ is a martingale and there exists a constant $K_n$ such that $|C_n(\omega)| \le K_n$. Then $(Y_n)_{n \ge 1}$ defined in \eqref{eqn:cumulative-win-process} is also a martingale. \end{lemma} \begin{remark} The assumption of $K_n$ being constant can be weakened to $C_n \in L^p(\bP)$, $X_n \in L^q(\bP)$ with $\frac{1}{p} + \frac{1}{q} = 1$. If $C_n \ge 0$ the assumption of $(X_n)_{n\ge 0}$ being a martingale can be weakened to it being a sub-/supermartingale. Then $(Y_n)_{n \ge 1}$ is a sub-/supermartingale as well. \end{remark} \begin{refproof}{lem:gambling-strategy} It is clear that $Y_n$ is $\cF_n$-measurable. Suppose that $C_n \in L^p(\bP)$ and $X_n \in L^{q}(\bP)$ for all $n$. We have \begin{IEEEeqnarray*}{rCl} \|Y_n\|_{L^1} &\le& \sum_{i=1}^{n} \|C_i(X_i - X_{i-1})\|_{L^1}\\ &\overset{\text{Hölder}}{\le}& \sum_{i=1}^{n} \|C_i\|_{L^p} \|(X_i - X_{i-1})\|_{L^q} \\ &<&\infty \end{IEEEeqnarray*} and \begin{IEEEeqnarray*}{rCl} \bE[Y_{n+1} - Y_n | \cF_n] &=& \bE[C_{n+1} (X_{n+1} - X_n) | \cF_n]\\ &=& C_{n+1} (\bE[X_{n+1} | \cF_n] - X_n)\\ &=& 0. \end{IEEEeqnarray*} \end{refproof} Suppose we have $(X_n)$ adapted, $X_n \in L^1(\bP)$, $(C_n)_{n \ge 1}$ previsible. We play according to the following principle: Pick two real numbers $a < b$. Wait until $X_n \le a$, then start playing. Stop playing when $X_n \ge b$. I.e.~define \begin{equation} \begin{aligned} C_1 &\coloneqq 0,\\ C_n &\coloneqq \One_{\{C_{n-1} = 1\}} \cdot \One_{\{X_{n-1} \le b\}} + \One_{\{C_{n-1} = 0\} } \One_{\{X_{n-1} < a\}}. \end{aligned} \label{eqn:upcrossing-strategy} \end{equation} \begin{definition} Fix $N \in \N$ and let \[U_N^X([a,b]) \coloneqq \# \{\text{Upcrossings of $[a,b]$ made by $n \mapsto X_n(\omega)$ by time $N$}\},\] i.e.~$U_N([a,b])(\omega)$ is the largest $k \in \N_0$ such that we can find a sequence $0 \le s_1 < t_1 < s_2 < t_2 < \ldots < s_k < t_k \le N$ such that $X_{s_j}(\omega) < a$ and $X_{t_j}(\omega) > b$ for all $1 \le j \le k$. \end{definition} Clearly $U_N^X([a,b]) \uparrow$ as $N$ increases. It follows that the monotonic limit \[U_\infty([a,b]) \coloneqq \lim_{N \to \infty} U_N([a,b])\] exists pointwise. \begin{lemma} % Lemma 1 \label{lec17l1} \[ \{\omega | \liminf_{N \to \infty} Z_N(\omega) < a < b < \limsup_{N \to \infty} Z_N(\omega)\} \subseteq \{\omega: U^{Z}_\infty([a,b])(\omega) = \infty\} \] for every sequence of measurable functions $(Z_n)_{n \ge 1}$. \end{lemma} \begin{lemma} % 2 \label{lec17l2} Let $Y_n(\omega) \coloneqq \sum_{j=1}^n C_j(X_j - X_{j-1})$, where $C_n $ is defined as in \eqref{eqn:upcrossing-strategy} Then \[Y_N \ge (b -a) U_N([a,b]) - (X_N - a)^{-}.\] \end{lemma} \begin{proof} Every upcrossing of $[a,b]$ increases the value of $Y$ by $(b-a)$, while the last interval of play $(X_n -a)^{-}$ overemphasizes the loss. \end{proof} \begin{lemma} %3 \label{lec17l3} Suppose $(X_n)_n$ is a supermartingale. Then in the above setup \[(b-a) \bE[U_N([a,b])] \le \bE[(X_N - a)^-].\] \end{lemma} \begin{proof} Since $C_n \ge 0$, by \yaref{lem:gambling-strategy} we have that $Y_n$ is a supermartingale. Hence $\bE[Y_N] \le \bE[Y_1] = 0$. From \yaref{lec17l2} it follows that \[ (b-a) \bE[U_N([a,b])] \le \bE[Y_n] + \bE[(X_N-a)^-] \le \bE[(X_N-a)^-]. \] \end{proof} \begin{corollary} Let $(X_n)_n$ be a \vocab[Supermartingale!bounded in $L^1$]{supermartingale bounded in $L^1(\bP)$}, i.e.~$\sup_n \bE[|X_n|] < \infty$. Then $(b-a) \bE(U_\infty) \le |a| + \sup_n \bE(|X_n|)$. In particular, $\bP[U_\infty = \infty] = 0$. \end{corollary} \begin{proof} By \yaref{lec17l3} we have that \[(b-a) \bE[U_N([a,b])] \le \bE[ | X_N| ] + |a| \le \sup_n \bE[|X_n|] + |a|.\] Since $U_N(\cdot) \ge 0$ and $U_N(\cdot ) \uparrow U_\infty(\cdot )$, by the monotone convergence theorem \[ \bE(U_N([a,b])] \uparrow \bE[U_\infty([a,b])]. \] \end{proof} Let us now consider the case that our process $(X_n)_{n \ge 1}$ is a supermartingale bounded in $L^1(\bP)$. \begin{theorem}[Doob's martingale convergence theorem] \yalabel{Doob's Martingale Convergence Theorem}{Doob}{doobmartingaleconvergence} \yalabel{Doob's Martingale Convergence Theorem}{Doob}{doob} Any supermartingale bounded in $L^1$ converges almost surely to a random variable, which is almost surely finite. In particular, any non-negative supermartingale converges a.s.~to a finite random variable. \end{theorem} \begin{refproof}{doobmartingaleconvergence} Let \[ \Lambda \coloneqq \{\omega | X_n(\omega) \text{ does not converge to anything in $[-\infty,\infty]$}\}. \] We have \begin{IEEEeqnarray*}{rCl} \Lambda &=& \{\omega | \liminf_N X_N(\omega) < \limsup_N X_N(\omega)\}\\ &=& \{\omega | \liminf_N X_N(\omega) < a < b < \limsup_N X_N(\omega)\} \\ &=& \bigcup_{a,b \in \Q} \underbrace{\{\omega | \liminf_N X_N(\omega) < a < b < \limsup_N X_N(\omega)\}}_{\Lambda_{a,b}} \\ \end{IEEEeqnarray*} We have $\Lambda_{a,b} \subseteq \{\omega : U_{\infty}([a,b])(\omega) = \infty\}$ by \yaref{lec17l1}. By \yaref{lec17l3} we have $\bP(\Lambda_{a,b}) = 0$, hence $\bP(\Lambda) = 0$. Thus there exists a random variable $X_\infty$ such that $X_n \xrightarrow{a.s.} X_\infty$. \begin{claim} $\bP[X_\infty \in \{\pm \infty\}] = 0$. \end{claim} \begin{subproof} It suffices to show that $\bE[|X_\infty|] < \infty$. We have. \begin{IEEEeqnarray*}{rCl} \bE[|X_\infty|] &=& \bE[\liminf_{n \to \infty} |X_n|]\\ &\overset{\yaref{cfatou}}{\le }& \liminf_n \bE[|X_n|]\\ &\le & \sup_n \bE[|X_n|]\\ &<& \infty. \end{IEEEeqnarray*} \end{subproof} The second part follows from \begin{claim} Any non-negative supermartingale is bounded in $L^1$. \end{claim} \begin{subproof} We need to show $\sup_n \bE(|X_n|) < \infty$. Since the supermartingale is non-negative, we have $\bE[|X_n|] = \bE[X_n]$ and since it is a supermartingale $\bE[X_n] \le \bE[X_0]$. \end{subproof} \end{refproof}